Find $int_0^1frac{ln^2(1-x)}{x} dx$












10












$begingroup$


In solving $displaystyleint_0^frac{pi}{4}dfrac{ln(sin x)ln(cos x)}{sin xcos x} dx,$ I have found that this is equal to $dfrac{1}{16}displaystyleint_0^1dfrac{ln^2(1-x)}{x} dx.$ WolframAlpha says that the desired value is $dfrac{zeta(3)}{8},$ so I suspect a conversion to a series is necessary.



How do I prove $displaystyleint_0^1dfrac{ln^2(1-x)}{x} dx=displaystylesum_{n=1}^inftydfrac{2}{n^3}$?



Note that the above integral can also be given as $displaystyleint_0^1dfrac{ln^2x}{1-x} dx$, which I know is equal to $displaystylesum_{n=0}^infty x^nln^2x.$



Also for reference, here is a picture of my original work to get to this point.



enter image description here










share|cite|improve this question











$endgroup$












  • $begingroup$
    What's up with everyone usign displaystyle lately? We don't need that anywhere, especially in the title
    $endgroup$
    – Yuriy S
    Aug 4 '16 at 21:01










  • $begingroup$
    @You'reInMyEye That's just how I write integrals. Chalk it up to me not having a lot of experience on Math.SE and not knowing all of the minutiae that you all prefer. My apologies.
    $endgroup$
    – Arcturus
    Aug 4 '16 at 21:09












  • $begingroup$
    Eridan, that was nothing personal, more a comment for general public. I edited at least three displaystyle titles today. I'm not sure myself why it's not allowed, but I think it may cause some problems with browsing question and search
    $endgroup$
    – Yuriy S
    Aug 4 '16 at 21:11












  • $begingroup$
    @You'reInMyEye MSE strongly recommends ( in some documentation ) to avoid $texttt{displaystyle}$ in the title.
    $endgroup$
    – Felix Marin
    Aug 4 '16 at 23:35










  • $begingroup$
    Is $ln^2 x$ a standard notation?
    $endgroup$
    – StubbornAtom
    Aug 5 '16 at 4:48
















10












$begingroup$


In solving $displaystyleint_0^frac{pi}{4}dfrac{ln(sin x)ln(cos x)}{sin xcos x} dx,$ I have found that this is equal to $dfrac{1}{16}displaystyleint_0^1dfrac{ln^2(1-x)}{x} dx.$ WolframAlpha says that the desired value is $dfrac{zeta(3)}{8},$ so I suspect a conversion to a series is necessary.



How do I prove $displaystyleint_0^1dfrac{ln^2(1-x)}{x} dx=displaystylesum_{n=1}^inftydfrac{2}{n^3}$?



Note that the above integral can also be given as $displaystyleint_0^1dfrac{ln^2x}{1-x} dx$, which I know is equal to $displaystylesum_{n=0}^infty x^nln^2x.$



Also for reference, here is a picture of my original work to get to this point.



enter image description here










share|cite|improve this question











$endgroup$












  • $begingroup$
    What's up with everyone usign displaystyle lately? We don't need that anywhere, especially in the title
    $endgroup$
    – Yuriy S
    Aug 4 '16 at 21:01










  • $begingroup$
    @You'reInMyEye That's just how I write integrals. Chalk it up to me not having a lot of experience on Math.SE and not knowing all of the minutiae that you all prefer. My apologies.
    $endgroup$
    – Arcturus
    Aug 4 '16 at 21:09












  • $begingroup$
    Eridan, that was nothing personal, more a comment for general public. I edited at least three displaystyle titles today. I'm not sure myself why it's not allowed, but I think it may cause some problems with browsing question and search
    $endgroup$
    – Yuriy S
    Aug 4 '16 at 21:11












  • $begingroup$
    @You'reInMyEye MSE strongly recommends ( in some documentation ) to avoid $texttt{displaystyle}$ in the title.
    $endgroup$
    – Felix Marin
    Aug 4 '16 at 23:35










  • $begingroup$
    Is $ln^2 x$ a standard notation?
    $endgroup$
    – StubbornAtom
    Aug 5 '16 at 4:48














10












10








10


3



$begingroup$


In solving $displaystyleint_0^frac{pi}{4}dfrac{ln(sin x)ln(cos x)}{sin xcos x} dx,$ I have found that this is equal to $dfrac{1}{16}displaystyleint_0^1dfrac{ln^2(1-x)}{x} dx.$ WolframAlpha says that the desired value is $dfrac{zeta(3)}{8},$ so I suspect a conversion to a series is necessary.



How do I prove $displaystyleint_0^1dfrac{ln^2(1-x)}{x} dx=displaystylesum_{n=1}^inftydfrac{2}{n^3}$?



Note that the above integral can also be given as $displaystyleint_0^1dfrac{ln^2x}{1-x} dx$, which I know is equal to $displaystylesum_{n=0}^infty x^nln^2x.$



Also for reference, here is a picture of my original work to get to this point.



enter image description here










share|cite|improve this question











$endgroup$




In solving $displaystyleint_0^frac{pi}{4}dfrac{ln(sin x)ln(cos x)}{sin xcos x} dx,$ I have found that this is equal to $dfrac{1}{16}displaystyleint_0^1dfrac{ln^2(1-x)}{x} dx.$ WolframAlpha says that the desired value is $dfrac{zeta(3)}{8},$ so I suspect a conversion to a series is necessary.



How do I prove $displaystyleint_0^1dfrac{ln^2(1-x)}{x} dx=displaystylesum_{n=1}^inftydfrac{2}{n^3}$?



Note that the above integral can also be given as $displaystyleint_0^1dfrac{ln^2x}{1-x} dx$, which I know is equal to $displaystylesum_{n=0}^infty x^nln^2x.$



Also for reference, here is a picture of my original work to get to this point.



enter image description here







calculus integration sequences-and-series definite-integrals






share|cite|improve this question















share|cite|improve this question













share|cite|improve this question




share|cite|improve this question








edited May 27 '18 at 4:32









Martin Sleziak

44.7k9117272




44.7k9117272










asked Aug 4 '16 at 20:31









ArcturusArcturus

536313




536313












  • $begingroup$
    What's up with everyone usign displaystyle lately? We don't need that anywhere, especially in the title
    $endgroup$
    – Yuriy S
    Aug 4 '16 at 21:01










  • $begingroup$
    @You'reInMyEye That's just how I write integrals. Chalk it up to me not having a lot of experience on Math.SE and not knowing all of the minutiae that you all prefer. My apologies.
    $endgroup$
    – Arcturus
    Aug 4 '16 at 21:09












  • $begingroup$
    Eridan, that was nothing personal, more a comment for general public. I edited at least three displaystyle titles today. I'm not sure myself why it's not allowed, but I think it may cause some problems with browsing question and search
    $endgroup$
    – Yuriy S
    Aug 4 '16 at 21:11












  • $begingroup$
    @You'reInMyEye MSE strongly recommends ( in some documentation ) to avoid $texttt{displaystyle}$ in the title.
    $endgroup$
    – Felix Marin
    Aug 4 '16 at 23:35










  • $begingroup$
    Is $ln^2 x$ a standard notation?
    $endgroup$
    – StubbornAtom
    Aug 5 '16 at 4:48


















  • $begingroup$
    What's up with everyone usign displaystyle lately? We don't need that anywhere, especially in the title
    $endgroup$
    – Yuriy S
    Aug 4 '16 at 21:01










  • $begingroup$
    @You'reInMyEye That's just how I write integrals. Chalk it up to me not having a lot of experience on Math.SE and not knowing all of the minutiae that you all prefer. My apologies.
    $endgroup$
    – Arcturus
    Aug 4 '16 at 21:09












  • $begingroup$
    Eridan, that was nothing personal, more a comment for general public. I edited at least three displaystyle titles today. I'm not sure myself why it's not allowed, but I think it may cause some problems with browsing question and search
    $endgroup$
    – Yuriy S
    Aug 4 '16 at 21:11












  • $begingroup$
    @You'reInMyEye MSE strongly recommends ( in some documentation ) to avoid $texttt{displaystyle}$ in the title.
    $endgroup$
    – Felix Marin
    Aug 4 '16 at 23:35










  • $begingroup$
    Is $ln^2 x$ a standard notation?
    $endgroup$
    – StubbornAtom
    Aug 5 '16 at 4:48
















$begingroup$
What's up with everyone usign displaystyle lately? We don't need that anywhere, especially in the title
$endgroup$
– Yuriy S
Aug 4 '16 at 21:01




$begingroup$
What's up with everyone usign displaystyle lately? We don't need that anywhere, especially in the title
$endgroup$
– Yuriy S
Aug 4 '16 at 21:01












$begingroup$
@You'reInMyEye That's just how I write integrals. Chalk it up to me not having a lot of experience on Math.SE and not knowing all of the minutiae that you all prefer. My apologies.
$endgroup$
– Arcturus
Aug 4 '16 at 21:09






$begingroup$
@You'reInMyEye That's just how I write integrals. Chalk it up to me not having a lot of experience on Math.SE and not knowing all of the minutiae that you all prefer. My apologies.
$endgroup$
– Arcturus
Aug 4 '16 at 21:09














$begingroup$
Eridan, that was nothing personal, more a comment for general public. I edited at least three displaystyle titles today. I'm not sure myself why it's not allowed, but I think it may cause some problems with browsing question and search
$endgroup$
– Yuriy S
Aug 4 '16 at 21:11






$begingroup$
Eridan, that was nothing personal, more a comment for general public. I edited at least three displaystyle titles today. I'm not sure myself why it's not allowed, but I think it may cause some problems with browsing question and search
$endgroup$
– Yuriy S
Aug 4 '16 at 21:11














$begingroup$
@You'reInMyEye MSE strongly recommends ( in some documentation ) to avoid $texttt{displaystyle}$ in the title.
$endgroup$
– Felix Marin
Aug 4 '16 at 23:35




$begingroup$
@You'reInMyEye MSE strongly recommends ( in some documentation ) to avoid $texttt{displaystyle}$ in the title.
$endgroup$
– Felix Marin
Aug 4 '16 at 23:35












$begingroup$
Is $ln^2 x$ a standard notation?
$endgroup$
– StubbornAtom
Aug 5 '16 at 4:48




$begingroup$
Is $ln^2 x$ a standard notation?
$endgroup$
– StubbornAtom
Aug 5 '16 at 4:48










5 Answers
5






active

oldest

votes


















11












$begingroup$

We have $$int_{0}^{1}frac{log^{2}left(1-xright)}{x}dxstackrel{xrightarrow1-x}{=}int_{0}^{1}frac{log^{2}left(xright)}{1-x}dx$$ $$stackrel{DCT}{=}
sum_{kgeq0}int_{0}^{1}log^{2}left(xright)x^{k}dxstackrel{IBP}{=}
2sum_{kgeq0}frac{1}{left(k+1right)^{3}}=color{red}{2zetaleft(3right)}.$$






share|cite|improve this answer











$endgroup$













  • $begingroup$
    Can you please tell me what $DCT$ means.
    $endgroup$
    – Ahmed S. Attaalla
    Aug 4 '16 at 23:44








  • 1




    $begingroup$
    @AhmedS.Attaalla I believe it's the dominated convergence theorem which allows him to switch the integral sign with the sum, since he used a geometric series representation of $frac{1}{1-x}$.
    $endgroup$
    – mike van der naald
    Aug 5 '16 at 0:14



















3












$begingroup$

I thought it might be instructive to present a way forward that exploits the Polylogarithm Functions. To that end, we proceed.



Note that integrating by parts with $u=log^2(1-x)$ and $v=log(x)$, we have



$$begin{align}
int_0^1 frac{log^2(1-x)}{x},dx=2int_0^1 frac{log(1-x)log(x)}{1-x},dx tag 1
end{align}$$



Integrating by parts the right-hand side of $(1)$ with $u=log(1-x)$ and $v=text{Li}_2(1-x)$ yields



$$begin{align}
2int_0^1 frac{log(1-x)log(x)}{1-x},dx&=2int_0^1 frac{text{Li}_2(1-x)}{1-x},dx\\
&=2int_0^1 frac{text{Li}_2(x)}{x},dx\\
&=2text{Li}_3(1)\\
&=2zeta(3)
end{align}$$



as expected!






share|cite|improve this answer









$endgroup$





















    2












    $begingroup$

    $newcommand{angles}[1]{leftlangle,{#1},rightrangle}
    newcommand{braces}[1]{leftlbrace,{#1},rightrbrace}
    newcommand{bracks}[1]{leftlbrack,{#1},rightrbrack}
    newcommand{dd}{mathrm{d}}
    newcommand{ds}[1]{displaystyle{#1}}
    newcommand{expo}[1]{,mathrm{e}^{#1},}
    newcommand{half}{{1 over 2}}
    newcommand{ic}{mathrm{i}}
    newcommand{iff}{Longleftrightarrow}
    newcommand{imp}{Longrightarrow}
    newcommand{Li}[1]{,mathrm{Li}_{#1}}
    newcommand{mc}[1]{,mathcal{#1}}
    newcommand{mrm}[1]{,mathrm{#1}}
    newcommand{ol}[1]{overline{#1}}
    newcommand{pars}[1]{left(,{#1},right)}
    newcommand{partiald}[3]{frac{partial^{#1} #2}{partial #3^{#1}}}
    newcommand{root}[2]{,sqrt[#1]{,{#2},},}
    newcommand{totald}[3]{frac{mathrm{d}^{#1} #2}{mathrm{d} #3^{#1}}}
    newcommand{ul}[1]{underline{#1}}
    newcommand{verts}[1]{leftvert,{#1},rightvert}$




    1. This one is $ul{slightly different}$ of the straightforward @Dr. MV answer:
      begin{align}
      color{#f00}{{1 over 16}int_{0}^{1}{ln^{2}pars{1 - x} over x},dd x} &,,,
      stackrel{x mapsto pars{1 - x}}{=},,,
      {1 over 16}int_{0}^{1}{ln^{2}pars{x} over 1 - x},dd x
      end{align}


      Integrating by Parts a few times ( the main purpose is to 'sit' a
      $ds{lnpars{1 - x}}$-factor in the integrand numerator ):
      begin{align}
      color{#f00}{{1 over 16}int_{0}^{1}{ln^{2}pars{1 - x} over x},dd x} & =
      {1 over 16}int_{0}^{1}lnpars{1 - x}
      bracks{2lnpars{x},{1 over x}},dd x =
      -,{1 over 8}int_{0}^{1}Li{2}'pars{x}lnpars{x},dd x
      \[5mm] & =
      {1 over 8}int_{0}^{1}Li{2}pars{x},{1 over x},dd x =
      {1 over 8}int_{0}^{1}Li{3}'pars{x},dd x = {1 over 8},Li{3}pars{1}
      \[5mm] & = color{#f00}{{1 over 8},zetapars{3}}
      end{align}

    2. Another approach uses the Beta Function
      $ds{mrm{B}pars{mu,nu} =
      int_{0}^{1}x^{mu - 1},pars{1 - x}^{nu - 1},,dd x =
      {Gammapars{mu}Gammapars{nu} over Gammapars{mu + nu}}}$ with
      $ds{Repars{mu} > 0,, Repars{nu} > 0}$. $ds{Gamma,}$: Gamma
      Function
      .
      begin{align}
      &color{#f00}{{1 over 16}int_{0}^{1}{ln^{2}pars{1 - x} over x},dd x} =
      {1 over 16},lim_{mu to 0},,partiald[2]{}{mu}
      int_{0}^{1}{pars{1 - x}^{mu} - 1 over x},dd x
      \[5mm] & =
      {1 over 16},lim_{mu to 0},,partiald[2]{}{mu}bracks{mu
      int_{0}^{1}lnpars{x}pars{1 - x}^{mu - 1},dd x} =
      {1 over 16},lim_{mu to 0 atop nu to 0},,{partial^{3} over partialmu^{2},partialnu}
      bracks{muint_{0}^{1}x^{nu}pars{1 - x}^{mu - 1},dd x}
      \[5mm] & =
      {1 over 16},lim_{mu to 0 atop nu to 0},,{partial^{3} over partialmu^{2},partialnu}bracks{mu,{Gammapars{nu + 1}Gammapars{mu} over Gammapars{mu + nu + 1}}} =
      {1 over 16},lim_{mu to 0 atop nu to 0},,{partial^{3} over partialmu^{2},partialnu}bracks{Gammapars{nu + 1}Gammapars{mu + 1} over Gammapars{mu + nu + 1}}
      \[5mm] & = color{#f00}{{1 over 8},zetapars{3}}
      end{align}






    share|cite|improve this answer











    $endgroup$





















      2












      $begingroup$

      Here is an approach that makes use of an Euler sum.



      We will first find a Maclaurin series expansion for $ln^2 (1 - x)$. As
      $$ln (1 - x) = - sum_{n = 1}^infty frac{x^n}{n},$$
      we have
      begin{align*}
      ln^2 (1 - x) &= left (- sum_{n = 1}^infty frac{x^n}{n} right ) cdot left (- sum_{n = 1}^infty frac{x^n}{n} right ).
      end{align*}

      Shifting the summation index $n mapsto n + 1$ gives
      begin{align*}
      ln^2 (1 - x) &= x^2 left (- sum_{n = 0}^infty frac{x^n}{n + 1} right ) cdot left (- sum_{n = 0}^infty frac{x^n}{n + 1} right )\
      &= sum_{n = 0}^infty sum_{k = 0}^n frac{x^{n + 2}}{(k + 1)(n - k + 1)},
      end{align*}

      where the last line has been obtained by applying the Cauchy product.



      Shifting the summation indices as follows: $n mapsto n - 2, k mapsto k - 1$ gives
      begin{align*}
      ln^2 (1 - x) &= sum_{n = 2}^infty sum_{k = 1}^{n - 1} frac{x^n}{k(n - k)}\
      &= sum_{n = 2}^infty sum_{k = 1}^{n - 1} left (frac{1}{nk} + frac{1}{n(n - k)} right ) x^n\
      &= 2 sum_{n = 2}^infty frac{x^n}{n} sum_{k = 1}^{n - 1} frac{1}{k}\
      &= 2 sum_{n = 2}^infty frac{H_{n - 1} x^n}{n},
      end{align*}

      where $H_n$ is the $n$th harmonic number.



      Now evaluating the integral. From the above Maclaurin series expansion for $ln^2 (1 - x)$ the integral can be written as
      begin{align*}
      int_0^1 frac{ln^2 (1 - x)}{x} , dx &= 2 sum_{n = 2}^infty frac{H_{n - 1}}{n} int_0^1 x^{n - 1} , dx = 2 sum_{n = 2}^infty frac{H_{n - 1}}{n^2}.
      end{align*}



      From properties of the harmonic numbers we have
      $$H_n = H_{n - 1} + frac{1}{n},$$
      thus
      begin{align*}
      int_0^1 frac{ln^2 (1 - x)}{x} , dx &= 2 sum_{n = 2}^infty frac{H_n}{n^2} - 2 sum_{n = 2}^infty frac{1}{n^3} = 2 sum_{n = 1}^infty frac{H_n}{n^2} - 2 sum_{n = 1}^infty frac{1}{n^3}.
      end{align*}



      Each sum can be readily found. They are:
      $$sum_{n = 1}^infty frac{1}{n^3} = zeta (3) quad text{and} quad sum_{n = 1}^infty frac{H_n}{n^2} = 2 zeta (3).$$
      A proof of the result for the second sum containing the harmonic number can, for example, be found here. Thus
      $$int_0^1 frac{ln^2 (1 - x)}{x} , dx = 4 zeta (3) - 2 zeta (3) = 2 zeta (3),$$
      as required.






      share|cite|improve this answer











      $endgroup$





















        0












        $begingroup$

        Everything you did is good. Indeed, it suffices to show that $I_n:=int_0^1 x^n ln^2 x = frac{2}{(n+1)^3}$ to conclude. The interversion $int / Sigma$ is possible as everything here is positive.



        To compute $I_n$, I tried an integration by parts (using that a primitive of $ln^2 x$ is $x(ln^2 x-2ln x)$) to obtain the relation $$I_n = frac{-2}{n+1}int_0^1 x^nln x.$$
        Let us call this latter integral $J_n$. Once again with an integration by parts, you can show that $J_n = frac{-1}{(n+1)^2}$, and thus conclude.






        share|cite|improve this answer









        $endgroup$













          Your Answer





          StackExchange.ifUsing("editor", function () {
          return StackExchange.using("mathjaxEditing", function () {
          StackExchange.MarkdownEditor.creationCallbacks.add(function (editor, postfix) {
          StackExchange.mathjaxEditing.prepareWmdForMathJax(editor, postfix, [["$", "$"], ["\\(","\\)"]]);
          });
          });
          }, "mathjax-editing");

          StackExchange.ready(function() {
          var channelOptions = {
          tags: "".split(" "),
          id: "69"
          };
          initTagRenderer("".split(" "), "".split(" "), channelOptions);

          StackExchange.using("externalEditor", function() {
          // Have to fire editor after snippets, if snippets enabled
          if (StackExchange.settings.snippets.snippetsEnabled) {
          StackExchange.using("snippets", function() {
          createEditor();
          });
          }
          else {
          createEditor();
          }
          });

          function createEditor() {
          StackExchange.prepareEditor({
          heartbeatType: 'answer',
          autoActivateHeartbeat: false,
          convertImagesToLinks: true,
          noModals: true,
          showLowRepImageUploadWarning: true,
          reputationToPostImages: 10,
          bindNavPrevention: true,
          postfix: "",
          imageUploader: {
          brandingHtml: "Powered by u003ca class="icon-imgur-white" href="https://imgur.com/"u003eu003c/au003e",
          contentPolicyHtml: "User contributions licensed under u003ca href="https://creativecommons.org/licenses/by-sa/3.0/"u003ecc by-sa 3.0 with attribution requiredu003c/au003e u003ca href="https://stackoverflow.com/legal/content-policy"u003e(content policy)u003c/au003e",
          allowUrls: true
          },
          noCode: true, onDemand: true,
          discardSelector: ".discard-answer"
          ,immediatelyShowMarkdownHelp:true
          });


          }
          });














          draft saved

          draft discarded


















          StackExchange.ready(
          function () {
          StackExchange.openid.initPostLogin('.new-post-login', 'https%3a%2f%2fmath.stackexchange.com%2fquestions%2f1882695%2ffind-int-01-frac-ln21-xx-dx%23new-answer', 'question_page');
          }
          );

          Post as a guest















          Required, but never shown

























          5 Answers
          5






          active

          oldest

          votes








          5 Answers
          5






          active

          oldest

          votes









          active

          oldest

          votes






          active

          oldest

          votes









          11












          $begingroup$

          We have $$int_{0}^{1}frac{log^{2}left(1-xright)}{x}dxstackrel{xrightarrow1-x}{=}int_{0}^{1}frac{log^{2}left(xright)}{1-x}dx$$ $$stackrel{DCT}{=}
          sum_{kgeq0}int_{0}^{1}log^{2}left(xright)x^{k}dxstackrel{IBP}{=}
          2sum_{kgeq0}frac{1}{left(k+1right)^{3}}=color{red}{2zetaleft(3right)}.$$






          share|cite|improve this answer











          $endgroup$













          • $begingroup$
            Can you please tell me what $DCT$ means.
            $endgroup$
            – Ahmed S. Attaalla
            Aug 4 '16 at 23:44








          • 1




            $begingroup$
            @AhmedS.Attaalla I believe it's the dominated convergence theorem which allows him to switch the integral sign with the sum, since he used a geometric series representation of $frac{1}{1-x}$.
            $endgroup$
            – mike van der naald
            Aug 5 '16 at 0:14
















          11












          $begingroup$

          We have $$int_{0}^{1}frac{log^{2}left(1-xright)}{x}dxstackrel{xrightarrow1-x}{=}int_{0}^{1}frac{log^{2}left(xright)}{1-x}dx$$ $$stackrel{DCT}{=}
          sum_{kgeq0}int_{0}^{1}log^{2}left(xright)x^{k}dxstackrel{IBP}{=}
          2sum_{kgeq0}frac{1}{left(k+1right)^{3}}=color{red}{2zetaleft(3right)}.$$






          share|cite|improve this answer











          $endgroup$













          • $begingroup$
            Can you please tell me what $DCT$ means.
            $endgroup$
            – Ahmed S. Attaalla
            Aug 4 '16 at 23:44








          • 1




            $begingroup$
            @AhmedS.Attaalla I believe it's the dominated convergence theorem which allows him to switch the integral sign with the sum, since he used a geometric series representation of $frac{1}{1-x}$.
            $endgroup$
            – mike van der naald
            Aug 5 '16 at 0:14














          11












          11








          11





          $begingroup$

          We have $$int_{0}^{1}frac{log^{2}left(1-xright)}{x}dxstackrel{xrightarrow1-x}{=}int_{0}^{1}frac{log^{2}left(xright)}{1-x}dx$$ $$stackrel{DCT}{=}
          sum_{kgeq0}int_{0}^{1}log^{2}left(xright)x^{k}dxstackrel{IBP}{=}
          2sum_{kgeq0}frac{1}{left(k+1right)^{3}}=color{red}{2zetaleft(3right)}.$$






          share|cite|improve this answer











          $endgroup$



          We have $$int_{0}^{1}frac{log^{2}left(1-xright)}{x}dxstackrel{xrightarrow1-x}{=}int_{0}^{1}frac{log^{2}left(xright)}{1-x}dx$$ $$stackrel{DCT}{=}
          sum_{kgeq0}int_{0}^{1}log^{2}left(xright)x^{k}dxstackrel{IBP}{=}
          2sum_{kgeq0}frac{1}{left(k+1right)^{3}}=color{red}{2zetaleft(3right)}.$$







          share|cite|improve this answer














          share|cite|improve this answer



          share|cite|improve this answer








          edited Aug 4 '16 at 20:53

























          answered Aug 4 '16 at 20:44









          Marco CantariniMarco Cantarini

          29.1k23373




          29.1k23373












          • $begingroup$
            Can you please tell me what $DCT$ means.
            $endgroup$
            – Ahmed S. Attaalla
            Aug 4 '16 at 23:44








          • 1




            $begingroup$
            @AhmedS.Attaalla I believe it's the dominated convergence theorem which allows him to switch the integral sign with the sum, since he used a geometric series representation of $frac{1}{1-x}$.
            $endgroup$
            – mike van der naald
            Aug 5 '16 at 0:14


















          • $begingroup$
            Can you please tell me what $DCT$ means.
            $endgroup$
            – Ahmed S. Attaalla
            Aug 4 '16 at 23:44








          • 1




            $begingroup$
            @AhmedS.Attaalla I believe it's the dominated convergence theorem which allows him to switch the integral sign with the sum, since he used a geometric series representation of $frac{1}{1-x}$.
            $endgroup$
            – mike van der naald
            Aug 5 '16 at 0:14
















          $begingroup$
          Can you please tell me what $DCT$ means.
          $endgroup$
          – Ahmed S. Attaalla
          Aug 4 '16 at 23:44






          $begingroup$
          Can you please tell me what $DCT$ means.
          $endgroup$
          – Ahmed S. Attaalla
          Aug 4 '16 at 23:44






          1




          1




          $begingroup$
          @AhmedS.Attaalla I believe it's the dominated convergence theorem which allows him to switch the integral sign with the sum, since he used a geometric series representation of $frac{1}{1-x}$.
          $endgroup$
          – mike van der naald
          Aug 5 '16 at 0:14




          $begingroup$
          @AhmedS.Attaalla I believe it's the dominated convergence theorem which allows him to switch the integral sign with the sum, since he used a geometric series representation of $frac{1}{1-x}$.
          $endgroup$
          – mike van der naald
          Aug 5 '16 at 0:14











          3












          $begingroup$

          I thought it might be instructive to present a way forward that exploits the Polylogarithm Functions. To that end, we proceed.



          Note that integrating by parts with $u=log^2(1-x)$ and $v=log(x)$, we have



          $$begin{align}
          int_0^1 frac{log^2(1-x)}{x},dx=2int_0^1 frac{log(1-x)log(x)}{1-x},dx tag 1
          end{align}$$



          Integrating by parts the right-hand side of $(1)$ with $u=log(1-x)$ and $v=text{Li}_2(1-x)$ yields



          $$begin{align}
          2int_0^1 frac{log(1-x)log(x)}{1-x},dx&=2int_0^1 frac{text{Li}_2(1-x)}{1-x},dx\\
          &=2int_0^1 frac{text{Li}_2(x)}{x},dx\\
          &=2text{Li}_3(1)\\
          &=2zeta(3)
          end{align}$$



          as expected!






          share|cite|improve this answer









          $endgroup$


















            3












            $begingroup$

            I thought it might be instructive to present a way forward that exploits the Polylogarithm Functions. To that end, we proceed.



            Note that integrating by parts with $u=log^2(1-x)$ and $v=log(x)$, we have



            $$begin{align}
            int_0^1 frac{log^2(1-x)}{x},dx=2int_0^1 frac{log(1-x)log(x)}{1-x},dx tag 1
            end{align}$$



            Integrating by parts the right-hand side of $(1)$ with $u=log(1-x)$ and $v=text{Li}_2(1-x)$ yields



            $$begin{align}
            2int_0^1 frac{log(1-x)log(x)}{1-x},dx&=2int_0^1 frac{text{Li}_2(1-x)}{1-x},dx\\
            &=2int_0^1 frac{text{Li}_2(x)}{x},dx\\
            &=2text{Li}_3(1)\\
            &=2zeta(3)
            end{align}$$



            as expected!






            share|cite|improve this answer









            $endgroup$
















              3












              3








              3





              $begingroup$

              I thought it might be instructive to present a way forward that exploits the Polylogarithm Functions. To that end, we proceed.



              Note that integrating by parts with $u=log^2(1-x)$ and $v=log(x)$, we have



              $$begin{align}
              int_0^1 frac{log^2(1-x)}{x},dx=2int_0^1 frac{log(1-x)log(x)}{1-x},dx tag 1
              end{align}$$



              Integrating by parts the right-hand side of $(1)$ with $u=log(1-x)$ and $v=text{Li}_2(1-x)$ yields



              $$begin{align}
              2int_0^1 frac{log(1-x)log(x)}{1-x},dx&=2int_0^1 frac{text{Li}_2(1-x)}{1-x},dx\\
              &=2int_0^1 frac{text{Li}_2(x)}{x},dx\\
              &=2text{Li}_3(1)\\
              &=2zeta(3)
              end{align}$$



              as expected!






              share|cite|improve this answer









              $endgroup$



              I thought it might be instructive to present a way forward that exploits the Polylogarithm Functions. To that end, we proceed.



              Note that integrating by parts with $u=log^2(1-x)$ and $v=log(x)$, we have



              $$begin{align}
              int_0^1 frac{log^2(1-x)}{x},dx=2int_0^1 frac{log(1-x)log(x)}{1-x},dx tag 1
              end{align}$$



              Integrating by parts the right-hand side of $(1)$ with $u=log(1-x)$ and $v=text{Li}_2(1-x)$ yields



              $$begin{align}
              2int_0^1 frac{log(1-x)log(x)}{1-x},dx&=2int_0^1 frac{text{Li}_2(1-x)}{1-x},dx\\
              &=2int_0^1 frac{text{Li}_2(x)}{x},dx\\
              &=2text{Li}_3(1)\\
              &=2zeta(3)
              end{align}$$



              as expected!







              share|cite|improve this answer












              share|cite|improve this answer



              share|cite|improve this answer










              answered Aug 4 '16 at 21:10









              Mark ViolaMark Viola

              131k1275171




              131k1275171























                  2












                  $begingroup$

                  $newcommand{angles}[1]{leftlangle,{#1},rightrangle}
                  newcommand{braces}[1]{leftlbrace,{#1},rightrbrace}
                  newcommand{bracks}[1]{leftlbrack,{#1},rightrbrack}
                  newcommand{dd}{mathrm{d}}
                  newcommand{ds}[1]{displaystyle{#1}}
                  newcommand{expo}[1]{,mathrm{e}^{#1},}
                  newcommand{half}{{1 over 2}}
                  newcommand{ic}{mathrm{i}}
                  newcommand{iff}{Longleftrightarrow}
                  newcommand{imp}{Longrightarrow}
                  newcommand{Li}[1]{,mathrm{Li}_{#1}}
                  newcommand{mc}[1]{,mathcal{#1}}
                  newcommand{mrm}[1]{,mathrm{#1}}
                  newcommand{ol}[1]{overline{#1}}
                  newcommand{pars}[1]{left(,{#1},right)}
                  newcommand{partiald}[3]{frac{partial^{#1} #2}{partial #3^{#1}}}
                  newcommand{root}[2]{,sqrt[#1]{,{#2},},}
                  newcommand{totald}[3]{frac{mathrm{d}^{#1} #2}{mathrm{d} #3^{#1}}}
                  newcommand{ul}[1]{underline{#1}}
                  newcommand{verts}[1]{leftvert,{#1},rightvert}$




                  1. This one is $ul{slightly different}$ of the straightforward @Dr. MV answer:
                    begin{align}
                    color{#f00}{{1 over 16}int_{0}^{1}{ln^{2}pars{1 - x} over x},dd x} &,,,
                    stackrel{x mapsto pars{1 - x}}{=},,,
                    {1 over 16}int_{0}^{1}{ln^{2}pars{x} over 1 - x},dd x
                    end{align}


                    Integrating by Parts a few times ( the main purpose is to 'sit' a
                    $ds{lnpars{1 - x}}$-factor in the integrand numerator ):
                    begin{align}
                    color{#f00}{{1 over 16}int_{0}^{1}{ln^{2}pars{1 - x} over x},dd x} & =
                    {1 over 16}int_{0}^{1}lnpars{1 - x}
                    bracks{2lnpars{x},{1 over x}},dd x =
                    -,{1 over 8}int_{0}^{1}Li{2}'pars{x}lnpars{x},dd x
                    \[5mm] & =
                    {1 over 8}int_{0}^{1}Li{2}pars{x},{1 over x},dd x =
                    {1 over 8}int_{0}^{1}Li{3}'pars{x},dd x = {1 over 8},Li{3}pars{1}
                    \[5mm] & = color{#f00}{{1 over 8},zetapars{3}}
                    end{align}

                  2. Another approach uses the Beta Function
                    $ds{mrm{B}pars{mu,nu} =
                    int_{0}^{1}x^{mu - 1},pars{1 - x}^{nu - 1},,dd x =
                    {Gammapars{mu}Gammapars{nu} over Gammapars{mu + nu}}}$ with
                    $ds{Repars{mu} > 0,, Repars{nu} > 0}$. $ds{Gamma,}$: Gamma
                    Function
                    .
                    begin{align}
                    &color{#f00}{{1 over 16}int_{0}^{1}{ln^{2}pars{1 - x} over x},dd x} =
                    {1 over 16},lim_{mu to 0},,partiald[2]{}{mu}
                    int_{0}^{1}{pars{1 - x}^{mu} - 1 over x},dd x
                    \[5mm] & =
                    {1 over 16},lim_{mu to 0},,partiald[2]{}{mu}bracks{mu
                    int_{0}^{1}lnpars{x}pars{1 - x}^{mu - 1},dd x} =
                    {1 over 16},lim_{mu to 0 atop nu to 0},,{partial^{3} over partialmu^{2},partialnu}
                    bracks{muint_{0}^{1}x^{nu}pars{1 - x}^{mu - 1},dd x}
                    \[5mm] & =
                    {1 over 16},lim_{mu to 0 atop nu to 0},,{partial^{3} over partialmu^{2},partialnu}bracks{mu,{Gammapars{nu + 1}Gammapars{mu} over Gammapars{mu + nu + 1}}} =
                    {1 over 16},lim_{mu to 0 atop nu to 0},,{partial^{3} over partialmu^{2},partialnu}bracks{Gammapars{nu + 1}Gammapars{mu + 1} over Gammapars{mu + nu + 1}}
                    \[5mm] & = color{#f00}{{1 over 8},zetapars{3}}
                    end{align}






                  share|cite|improve this answer











                  $endgroup$


















                    2












                    $begingroup$

                    $newcommand{angles}[1]{leftlangle,{#1},rightrangle}
                    newcommand{braces}[1]{leftlbrace,{#1},rightrbrace}
                    newcommand{bracks}[1]{leftlbrack,{#1},rightrbrack}
                    newcommand{dd}{mathrm{d}}
                    newcommand{ds}[1]{displaystyle{#1}}
                    newcommand{expo}[1]{,mathrm{e}^{#1},}
                    newcommand{half}{{1 over 2}}
                    newcommand{ic}{mathrm{i}}
                    newcommand{iff}{Longleftrightarrow}
                    newcommand{imp}{Longrightarrow}
                    newcommand{Li}[1]{,mathrm{Li}_{#1}}
                    newcommand{mc}[1]{,mathcal{#1}}
                    newcommand{mrm}[1]{,mathrm{#1}}
                    newcommand{ol}[1]{overline{#1}}
                    newcommand{pars}[1]{left(,{#1},right)}
                    newcommand{partiald}[3]{frac{partial^{#1} #2}{partial #3^{#1}}}
                    newcommand{root}[2]{,sqrt[#1]{,{#2},},}
                    newcommand{totald}[3]{frac{mathrm{d}^{#1} #2}{mathrm{d} #3^{#1}}}
                    newcommand{ul}[1]{underline{#1}}
                    newcommand{verts}[1]{leftvert,{#1},rightvert}$




                    1. This one is $ul{slightly different}$ of the straightforward @Dr. MV answer:
                      begin{align}
                      color{#f00}{{1 over 16}int_{0}^{1}{ln^{2}pars{1 - x} over x},dd x} &,,,
                      stackrel{x mapsto pars{1 - x}}{=},,,
                      {1 over 16}int_{0}^{1}{ln^{2}pars{x} over 1 - x},dd x
                      end{align}


                      Integrating by Parts a few times ( the main purpose is to 'sit' a
                      $ds{lnpars{1 - x}}$-factor in the integrand numerator ):
                      begin{align}
                      color{#f00}{{1 over 16}int_{0}^{1}{ln^{2}pars{1 - x} over x},dd x} & =
                      {1 over 16}int_{0}^{1}lnpars{1 - x}
                      bracks{2lnpars{x},{1 over x}},dd x =
                      -,{1 over 8}int_{0}^{1}Li{2}'pars{x}lnpars{x},dd x
                      \[5mm] & =
                      {1 over 8}int_{0}^{1}Li{2}pars{x},{1 over x},dd x =
                      {1 over 8}int_{0}^{1}Li{3}'pars{x},dd x = {1 over 8},Li{3}pars{1}
                      \[5mm] & = color{#f00}{{1 over 8},zetapars{3}}
                      end{align}

                    2. Another approach uses the Beta Function
                      $ds{mrm{B}pars{mu,nu} =
                      int_{0}^{1}x^{mu - 1},pars{1 - x}^{nu - 1},,dd x =
                      {Gammapars{mu}Gammapars{nu} over Gammapars{mu + nu}}}$ with
                      $ds{Repars{mu} > 0,, Repars{nu} > 0}$. $ds{Gamma,}$: Gamma
                      Function
                      .
                      begin{align}
                      &color{#f00}{{1 over 16}int_{0}^{1}{ln^{2}pars{1 - x} over x},dd x} =
                      {1 over 16},lim_{mu to 0},,partiald[2]{}{mu}
                      int_{0}^{1}{pars{1 - x}^{mu} - 1 over x},dd x
                      \[5mm] & =
                      {1 over 16},lim_{mu to 0},,partiald[2]{}{mu}bracks{mu
                      int_{0}^{1}lnpars{x}pars{1 - x}^{mu - 1},dd x} =
                      {1 over 16},lim_{mu to 0 atop nu to 0},,{partial^{3} over partialmu^{2},partialnu}
                      bracks{muint_{0}^{1}x^{nu}pars{1 - x}^{mu - 1},dd x}
                      \[5mm] & =
                      {1 over 16},lim_{mu to 0 atop nu to 0},,{partial^{3} over partialmu^{2},partialnu}bracks{mu,{Gammapars{nu + 1}Gammapars{mu} over Gammapars{mu + nu + 1}}} =
                      {1 over 16},lim_{mu to 0 atop nu to 0},,{partial^{3} over partialmu^{2},partialnu}bracks{Gammapars{nu + 1}Gammapars{mu + 1} over Gammapars{mu + nu + 1}}
                      \[5mm] & = color{#f00}{{1 over 8},zetapars{3}}
                      end{align}






                    share|cite|improve this answer











                    $endgroup$
















                      2












                      2








                      2





                      $begingroup$

                      $newcommand{angles}[1]{leftlangle,{#1},rightrangle}
                      newcommand{braces}[1]{leftlbrace,{#1},rightrbrace}
                      newcommand{bracks}[1]{leftlbrack,{#1},rightrbrack}
                      newcommand{dd}{mathrm{d}}
                      newcommand{ds}[1]{displaystyle{#1}}
                      newcommand{expo}[1]{,mathrm{e}^{#1},}
                      newcommand{half}{{1 over 2}}
                      newcommand{ic}{mathrm{i}}
                      newcommand{iff}{Longleftrightarrow}
                      newcommand{imp}{Longrightarrow}
                      newcommand{Li}[1]{,mathrm{Li}_{#1}}
                      newcommand{mc}[1]{,mathcal{#1}}
                      newcommand{mrm}[1]{,mathrm{#1}}
                      newcommand{ol}[1]{overline{#1}}
                      newcommand{pars}[1]{left(,{#1},right)}
                      newcommand{partiald}[3]{frac{partial^{#1} #2}{partial #3^{#1}}}
                      newcommand{root}[2]{,sqrt[#1]{,{#2},},}
                      newcommand{totald}[3]{frac{mathrm{d}^{#1} #2}{mathrm{d} #3^{#1}}}
                      newcommand{ul}[1]{underline{#1}}
                      newcommand{verts}[1]{leftvert,{#1},rightvert}$




                      1. This one is $ul{slightly different}$ of the straightforward @Dr. MV answer:
                        begin{align}
                        color{#f00}{{1 over 16}int_{0}^{1}{ln^{2}pars{1 - x} over x},dd x} &,,,
                        stackrel{x mapsto pars{1 - x}}{=},,,
                        {1 over 16}int_{0}^{1}{ln^{2}pars{x} over 1 - x},dd x
                        end{align}


                        Integrating by Parts a few times ( the main purpose is to 'sit' a
                        $ds{lnpars{1 - x}}$-factor in the integrand numerator ):
                        begin{align}
                        color{#f00}{{1 over 16}int_{0}^{1}{ln^{2}pars{1 - x} over x},dd x} & =
                        {1 over 16}int_{0}^{1}lnpars{1 - x}
                        bracks{2lnpars{x},{1 over x}},dd x =
                        -,{1 over 8}int_{0}^{1}Li{2}'pars{x}lnpars{x},dd x
                        \[5mm] & =
                        {1 over 8}int_{0}^{1}Li{2}pars{x},{1 over x},dd x =
                        {1 over 8}int_{0}^{1}Li{3}'pars{x},dd x = {1 over 8},Li{3}pars{1}
                        \[5mm] & = color{#f00}{{1 over 8},zetapars{3}}
                        end{align}

                      2. Another approach uses the Beta Function
                        $ds{mrm{B}pars{mu,nu} =
                        int_{0}^{1}x^{mu - 1},pars{1 - x}^{nu - 1},,dd x =
                        {Gammapars{mu}Gammapars{nu} over Gammapars{mu + nu}}}$ with
                        $ds{Repars{mu} > 0,, Repars{nu} > 0}$. $ds{Gamma,}$: Gamma
                        Function
                        .
                        begin{align}
                        &color{#f00}{{1 over 16}int_{0}^{1}{ln^{2}pars{1 - x} over x},dd x} =
                        {1 over 16},lim_{mu to 0},,partiald[2]{}{mu}
                        int_{0}^{1}{pars{1 - x}^{mu} - 1 over x},dd x
                        \[5mm] & =
                        {1 over 16},lim_{mu to 0},,partiald[2]{}{mu}bracks{mu
                        int_{0}^{1}lnpars{x}pars{1 - x}^{mu - 1},dd x} =
                        {1 over 16},lim_{mu to 0 atop nu to 0},,{partial^{3} over partialmu^{2},partialnu}
                        bracks{muint_{0}^{1}x^{nu}pars{1 - x}^{mu - 1},dd x}
                        \[5mm] & =
                        {1 over 16},lim_{mu to 0 atop nu to 0},,{partial^{3} over partialmu^{2},partialnu}bracks{mu,{Gammapars{nu + 1}Gammapars{mu} over Gammapars{mu + nu + 1}}} =
                        {1 over 16},lim_{mu to 0 atop nu to 0},,{partial^{3} over partialmu^{2},partialnu}bracks{Gammapars{nu + 1}Gammapars{mu + 1} over Gammapars{mu + nu + 1}}
                        \[5mm] & = color{#f00}{{1 over 8},zetapars{3}}
                        end{align}






                      share|cite|improve this answer











                      $endgroup$



                      $newcommand{angles}[1]{leftlangle,{#1},rightrangle}
                      newcommand{braces}[1]{leftlbrace,{#1},rightrbrace}
                      newcommand{bracks}[1]{leftlbrack,{#1},rightrbrack}
                      newcommand{dd}{mathrm{d}}
                      newcommand{ds}[1]{displaystyle{#1}}
                      newcommand{expo}[1]{,mathrm{e}^{#1},}
                      newcommand{half}{{1 over 2}}
                      newcommand{ic}{mathrm{i}}
                      newcommand{iff}{Longleftrightarrow}
                      newcommand{imp}{Longrightarrow}
                      newcommand{Li}[1]{,mathrm{Li}_{#1}}
                      newcommand{mc}[1]{,mathcal{#1}}
                      newcommand{mrm}[1]{,mathrm{#1}}
                      newcommand{ol}[1]{overline{#1}}
                      newcommand{pars}[1]{left(,{#1},right)}
                      newcommand{partiald}[3]{frac{partial^{#1} #2}{partial #3^{#1}}}
                      newcommand{root}[2]{,sqrt[#1]{,{#2},},}
                      newcommand{totald}[3]{frac{mathrm{d}^{#1} #2}{mathrm{d} #3^{#1}}}
                      newcommand{ul}[1]{underline{#1}}
                      newcommand{verts}[1]{leftvert,{#1},rightvert}$




                      1. This one is $ul{slightly different}$ of the straightforward @Dr. MV answer:
                        begin{align}
                        color{#f00}{{1 over 16}int_{0}^{1}{ln^{2}pars{1 - x} over x},dd x} &,,,
                        stackrel{x mapsto pars{1 - x}}{=},,,
                        {1 over 16}int_{0}^{1}{ln^{2}pars{x} over 1 - x},dd x
                        end{align}


                        Integrating by Parts a few times ( the main purpose is to 'sit' a
                        $ds{lnpars{1 - x}}$-factor in the integrand numerator ):
                        begin{align}
                        color{#f00}{{1 over 16}int_{0}^{1}{ln^{2}pars{1 - x} over x},dd x} & =
                        {1 over 16}int_{0}^{1}lnpars{1 - x}
                        bracks{2lnpars{x},{1 over x}},dd x =
                        -,{1 over 8}int_{0}^{1}Li{2}'pars{x}lnpars{x},dd x
                        \[5mm] & =
                        {1 over 8}int_{0}^{1}Li{2}pars{x},{1 over x},dd x =
                        {1 over 8}int_{0}^{1}Li{3}'pars{x},dd x = {1 over 8},Li{3}pars{1}
                        \[5mm] & = color{#f00}{{1 over 8},zetapars{3}}
                        end{align}

                      2. Another approach uses the Beta Function
                        $ds{mrm{B}pars{mu,nu} =
                        int_{0}^{1}x^{mu - 1},pars{1 - x}^{nu - 1},,dd x =
                        {Gammapars{mu}Gammapars{nu} over Gammapars{mu + nu}}}$ with
                        $ds{Repars{mu} > 0,, Repars{nu} > 0}$. $ds{Gamma,}$: Gamma
                        Function
                        .
                        begin{align}
                        &color{#f00}{{1 over 16}int_{0}^{1}{ln^{2}pars{1 - x} over x},dd x} =
                        {1 over 16},lim_{mu to 0},,partiald[2]{}{mu}
                        int_{0}^{1}{pars{1 - x}^{mu} - 1 over x},dd x
                        \[5mm] & =
                        {1 over 16},lim_{mu to 0},,partiald[2]{}{mu}bracks{mu
                        int_{0}^{1}lnpars{x}pars{1 - x}^{mu - 1},dd x} =
                        {1 over 16},lim_{mu to 0 atop nu to 0},,{partial^{3} over partialmu^{2},partialnu}
                        bracks{muint_{0}^{1}x^{nu}pars{1 - x}^{mu - 1},dd x}
                        \[5mm] & =
                        {1 over 16},lim_{mu to 0 atop nu to 0},,{partial^{3} over partialmu^{2},partialnu}bracks{mu,{Gammapars{nu + 1}Gammapars{mu} over Gammapars{mu + nu + 1}}} =
                        {1 over 16},lim_{mu to 0 atop nu to 0},,{partial^{3} over partialmu^{2},partialnu}bracks{Gammapars{nu + 1}Gammapars{mu + 1} over Gammapars{mu + nu + 1}}
                        \[5mm] & = color{#f00}{{1 over 8},zetapars{3}}
                        end{align}







                      share|cite|improve this answer














                      share|cite|improve this answer



                      share|cite|improve this answer








                      edited Apr 13 '17 at 12:21









                      Community

                      1




                      1










                      answered Aug 4 '16 at 22:49









                      Felix MarinFelix Marin

                      67.5k7107141




                      67.5k7107141























                          2












                          $begingroup$

                          Here is an approach that makes use of an Euler sum.



                          We will first find a Maclaurin series expansion for $ln^2 (1 - x)$. As
                          $$ln (1 - x) = - sum_{n = 1}^infty frac{x^n}{n},$$
                          we have
                          begin{align*}
                          ln^2 (1 - x) &= left (- sum_{n = 1}^infty frac{x^n}{n} right ) cdot left (- sum_{n = 1}^infty frac{x^n}{n} right ).
                          end{align*}

                          Shifting the summation index $n mapsto n + 1$ gives
                          begin{align*}
                          ln^2 (1 - x) &= x^2 left (- sum_{n = 0}^infty frac{x^n}{n + 1} right ) cdot left (- sum_{n = 0}^infty frac{x^n}{n + 1} right )\
                          &= sum_{n = 0}^infty sum_{k = 0}^n frac{x^{n + 2}}{(k + 1)(n - k + 1)},
                          end{align*}

                          where the last line has been obtained by applying the Cauchy product.



                          Shifting the summation indices as follows: $n mapsto n - 2, k mapsto k - 1$ gives
                          begin{align*}
                          ln^2 (1 - x) &= sum_{n = 2}^infty sum_{k = 1}^{n - 1} frac{x^n}{k(n - k)}\
                          &= sum_{n = 2}^infty sum_{k = 1}^{n - 1} left (frac{1}{nk} + frac{1}{n(n - k)} right ) x^n\
                          &= 2 sum_{n = 2}^infty frac{x^n}{n} sum_{k = 1}^{n - 1} frac{1}{k}\
                          &= 2 sum_{n = 2}^infty frac{H_{n - 1} x^n}{n},
                          end{align*}

                          where $H_n$ is the $n$th harmonic number.



                          Now evaluating the integral. From the above Maclaurin series expansion for $ln^2 (1 - x)$ the integral can be written as
                          begin{align*}
                          int_0^1 frac{ln^2 (1 - x)}{x} , dx &= 2 sum_{n = 2}^infty frac{H_{n - 1}}{n} int_0^1 x^{n - 1} , dx = 2 sum_{n = 2}^infty frac{H_{n - 1}}{n^2}.
                          end{align*}



                          From properties of the harmonic numbers we have
                          $$H_n = H_{n - 1} + frac{1}{n},$$
                          thus
                          begin{align*}
                          int_0^1 frac{ln^2 (1 - x)}{x} , dx &= 2 sum_{n = 2}^infty frac{H_n}{n^2} - 2 sum_{n = 2}^infty frac{1}{n^3} = 2 sum_{n = 1}^infty frac{H_n}{n^2} - 2 sum_{n = 1}^infty frac{1}{n^3}.
                          end{align*}



                          Each sum can be readily found. They are:
                          $$sum_{n = 1}^infty frac{1}{n^3} = zeta (3) quad text{and} quad sum_{n = 1}^infty frac{H_n}{n^2} = 2 zeta (3).$$
                          A proof of the result for the second sum containing the harmonic number can, for example, be found here. Thus
                          $$int_0^1 frac{ln^2 (1 - x)}{x} , dx = 4 zeta (3) - 2 zeta (3) = 2 zeta (3),$$
                          as required.






                          share|cite|improve this answer











                          $endgroup$


















                            2












                            $begingroup$

                            Here is an approach that makes use of an Euler sum.



                            We will first find a Maclaurin series expansion for $ln^2 (1 - x)$. As
                            $$ln (1 - x) = - sum_{n = 1}^infty frac{x^n}{n},$$
                            we have
                            begin{align*}
                            ln^2 (1 - x) &= left (- sum_{n = 1}^infty frac{x^n}{n} right ) cdot left (- sum_{n = 1}^infty frac{x^n}{n} right ).
                            end{align*}

                            Shifting the summation index $n mapsto n + 1$ gives
                            begin{align*}
                            ln^2 (1 - x) &= x^2 left (- sum_{n = 0}^infty frac{x^n}{n + 1} right ) cdot left (- sum_{n = 0}^infty frac{x^n}{n + 1} right )\
                            &= sum_{n = 0}^infty sum_{k = 0}^n frac{x^{n + 2}}{(k + 1)(n - k + 1)},
                            end{align*}

                            where the last line has been obtained by applying the Cauchy product.



                            Shifting the summation indices as follows: $n mapsto n - 2, k mapsto k - 1$ gives
                            begin{align*}
                            ln^2 (1 - x) &= sum_{n = 2}^infty sum_{k = 1}^{n - 1} frac{x^n}{k(n - k)}\
                            &= sum_{n = 2}^infty sum_{k = 1}^{n - 1} left (frac{1}{nk} + frac{1}{n(n - k)} right ) x^n\
                            &= 2 sum_{n = 2}^infty frac{x^n}{n} sum_{k = 1}^{n - 1} frac{1}{k}\
                            &= 2 sum_{n = 2}^infty frac{H_{n - 1} x^n}{n},
                            end{align*}

                            where $H_n$ is the $n$th harmonic number.



                            Now evaluating the integral. From the above Maclaurin series expansion for $ln^2 (1 - x)$ the integral can be written as
                            begin{align*}
                            int_0^1 frac{ln^2 (1 - x)}{x} , dx &= 2 sum_{n = 2}^infty frac{H_{n - 1}}{n} int_0^1 x^{n - 1} , dx = 2 sum_{n = 2}^infty frac{H_{n - 1}}{n^2}.
                            end{align*}



                            From properties of the harmonic numbers we have
                            $$H_n = H_{n - 1} + frac{1}{n},$$
                            thus
                            begin{align*}
                            int_0^1 frac{ln^2 (1 - x)}{x} , dx &= 2 sum_{n = 2}^infty frac{H_n}{n^2} - 2 sum_{n = 2}^infty frac{1}{n^3} = 2 sum_{n = 1}^infty frac{H_n}{n^2} - 2 sum_{n = 1}^infty frac{1}{n^3}.
                            end{align*}



                            Each sum can be readily found. They are:
                            $$sum_{n = 1}^infty frac{1}{n^3} = zeta (3) quad text{and} quad sum_{n = 1}^infty frac{H_n}{n^2} = 2 zeta (3).$$
                            A proof of the result for the second sum containing the harmonic number can, for example, be found here. Thus
                            $$int_0^1 frac{ln^2 (1 - x)}{x} , dx = 4 zeta (3) - 2 zeta (3) = 2 zeta (3),$$
                            as required.






                            share|cite|improve this answer











                            $endgroup$
















                              2












                              2








                              2





                              $begingroup$

                              Here is an approach that makes use of an Euler sum.



                              We will first find a Maclaurin series expansion for $ln^2 (1 - x)$. As
                              $$ln (1 - x) = - sum_{n = 1}^infty frac{x^n}{n},$$
                              we have
                              begin{align*}
                              ln^2 (1 - x) &= left (- sum_{n = 1}^infty frac{x^n}{n} right ) cdot left (- sum_{n = 1}^infty frac{x^n}{n} right ).
                              end{align*}

                              Shifting the summation index $n mapsto n + 1$ gives
                              begin{align*}
                              ln^2 (1 - x) &= x^2 left (- sum_{n = 0}^infty frac{x^n}{n + 1} right ) cdot left (- sum_{n = 0}^infty frac{x^n}{n + 1} right )\
                              &= sum_{n = 0}^infty sum_{k = 0}^n frac{x^{n + 2}}{(k + 1)(n - k + 1)},
                              end{align*}

                              where the last line has been obtained by applying the Cauchy product.



                              Shifting the summation indices as follows: $n mapsto n - 2, k mapsto k - 1$ gives
                              begin{align*}
                              ln^2 (1 - x) &= sum_{n = 2}^infty sum_{k = 1}^{n - 1} frac{x^n}{k(n - k)}\
                              &= sum_{n = 2}^infty sum_{k = 1}^{n - 1} left (frac{1}{nk} + frac{1}{n(n - k)} right ) x^n\
                              &= 2 sum_{n = 2}^infty frac{x^n}{n} sum_{k = 1}^{n - 1} frac{1}{k}\
                              &= 2 sum_{n = 2}^infty frac{H_{n - 1} x^n}{n},
                              end{align*}

                              where $H_n$ is the $n$th harmonic number.



                              Now evaluating the integral. From the above Maclaurin series expansion for $ln^2 (1 - x)$ the integral can be written as
                              begin{align*}
                              int_0^1 frac{ln^2 (1 - x)}{x} , dx &= 2 sum_{n = 2}^infty frac{H_{n - 1}}{n} int_0^1 x^{n - 1} , dx = 2 sum_{n = 2}^infty frac{H_{n - 1}}{n^2}.
                              end{align*}



                              From properties of the harmonic numbers we have
                              $$H_n = H_{n - 1} + frac{1}{n},$$
                              thus
                              begin{align*}
                              int_0^1 frac{ln^2 (1 - x)}{x} , dx &= 2 sum_{n = 2}^infty frac{H_n}{n^2} - 2 sum_{n = 2}^infty frac{1}{n^3} = 2 sum_{n = 1}^infty frac{H_n}{n^2} - 2 sum_{n = 1}^infty frac{1}{n^3}.
                              end{align*}



                              Each sum can be readily found. They are:
                              $$sum_{n = 1}^infty frac{1}{n^3} = zeta (3) quad text{and} quad sum_{n = 1}^infty frac{H_n}{n^2} = 2 zeta (3).$$
                              A proof of the result for the second sum containing the harmonic number can, for example, be found here. Thus
                              $$int_0^1 frac{ln^2 (1 - x)}{x} , dx = 4 zeta (3) - 2 zeta (3) = 2 zeta (3),$$
                              as required.






                              share|cite|improve this answer











                              $endgroup$



                              Here is an approach that makes use of an Euler sum.



                              We will first find a Maclaurin series expansion for $ln^2 (1 - x)$. As
                              $$ln (1 - x) = - sum_{n = 1}^infty frac{x^n}{n},$$
                              we have
                              begin{align*}
                              ln^2 (1 - x) &= left (- sum_{n = 1}^infty frac{x^n}{n} right ) cdot left (- sum_{n = 1}^infty frac{x^n}{n} right ).
                              end{align*}

                              Shifting the summation index $n mapsto n + 1$ gives
                              begin{align*}
                              ln^2 (1 - x) &= x^2 left (- sum_{n = 0}^infty frac{x^n}{n + 1} right ) cdot left (- sum_{n = 0}^infty frac{x^n}{n + 1} right )\
                              &= sum_{n = 0}^infty sum_{k = 0}^n frac{x^{n + 2}}{(k + 1)(n - k + 1)},
                              end{align*}

                              where the last line has been obtained by applying the Cauchy product.



                              Shifting the summation indices as follows: $n mapsto n - 2, k mapsto k - 1$ gives
                              begin{align*}
                              ln^2 (1 - x) &= sum_{n = 2}^infty sum_{k = 1}^{n - 1} frac{x^n}{k(n - k)}\
                              &= sum_{n = 2}^infty sum_{k = 1}^{n - 1} left (frac{1}{nk} + frac{1}{n(n - k)} right ) x^n\
                              &= 2 sum_{n = 2}^infty frac{x^n}{n} sum_{k = 1}^{n - 1} frac{1}{k}\
                              &= 2 sum_{n = 2}^infty frac{H_{n - 1} x^n}{n},
                              end{align*}

                              where $H_n$ is the $n$th harmonic number.



                              Now evaluating the integral. From the above Maclaurin series expansion for $ln^2 (1 - x)$ the integral can be written as
                              begin{align*}
                              int_0^1 frac{ln^2 (1 - x)}{x} , dx &= 2 sum_{n = 2}^infty frac{H_{n - 1}}{n} int_0^1 x^{n - 1} , dx = 2 sum_{n = 2}^infty frac{H_{n - 1}}{n^2}.
                              end{align*}



                              From properties of the harmonic numbers we have
                              $$H_n = H_{n - 1} + frac{1}{n},$$
                              thus
                              begin{align*}
                              int_0^1 frac{ln^2 (1 - x)}{x} , dx &= 2 sum_{n = 2}^infty frac{H_n}{n^2} - 2 sum_{n = 2}^infty frac{1}{n^3} = 2 sum_{n = 1}^infty frac{H_n}{n^2} - 2 sum_{n = 1}^infty frac{1}{n^3}.
                              end{align*}



                              Each sum can be readily found. They are:
                              $$sum_{n = 1}^infty frac{1}{n^3} = zeta (3) quad text{and} quad sum_{n = 1}^infty frac{H_n}{n^2} = 2 zeta (3).$$
                              A proof of the result for the second sum containing the harmonic number can, for example, be found here. Thus
                              $$int_0^1 frac{ln^2 (1 - x)}{x} , dx = 4 zeta (3) - 2 zeta (3) = 2 zeta (3),$$
                              as required.







                              share|cite|improve this answer














                              share|cite|improve this answer



                              share|cite|improve this answer








                              edited Jan 11 at 11:21

























                              answered Jan 3 '18 at 3:29









                              omegadotomegadot

                              5,1872727




                              5,1872727























                                  0












                                  $begingroup$

                                  Everything you did is good. Indeed, it suffices to show that $I_n:=int_0^1 x^n ln^2 x = frac{2}{(n+1)^3}$ to conclude. The interversion $int / Sigma$ is possible as everything here is positive.



                                  To compute $I_n$, I tried an integration by parts (using that a primitive of $ln^2 x$ is $x(ln^2 x-2ln x)$) to obtain the relation $$I_n = frac{-2}{n+1}int_0^1 x^nln x.$$
                                  Let us call this latter integral $J_n$. Once again with an integration by parts, you can show that $J_n = frac{-1}{(n+1)^2}$, and thus conclude.






                                  share|cite|improve this answer









                                  $endgroup$


















                                    0












                                    $begingroup$

                                    Everything you did is good. Indeed, it suffices to show that $I_n:=int_0^1 x^n ln^2 x = frac{2}{(n+1)^3}$ to conclude. The interversion $int / Sigma$ is possible as everything here is positive.



                                    To compute $I_n$, I tried an integration by parts (using that a primitive of $ln^2 x$ is $x(ln^2 x-2ln x)$) to obtain the relation $$I_n = frac{-2}{n+1}int_0^1 x^nln x.$$
                                    Let us call this latter integral $J_n$. Once again with an integration by parts, you can show that $J_n = frac{-1}{(n+1)^2}$, and thus conclude.






                                    share|cite|improve this answer









                                    $endgroup$
















                                      0












                                      0








                                      0





                                      $begingroup$

                                      Everything you did is good. Indeed, it suffices to show that $I_n:=int_0^1 x^n ln^2 x = frac{2}{(n+1)^3}$ to conclude. The interversion $int / Sigma$ is possible as everything here is positive.



                                      To compute $I_n$, I tried an integration by parts (using that a primitive of $ln^2 x$ is $x(ln^2 x-2ln x)$) to obtain the relation $$I_n = frac{-2}{n+1}int_0^1 x^nln x.$$
                                      Let us call this latter integral $J_n$. Once again with an integration by parts, you can show that $J_n = frac{-1}{(n+1)^2}$, and thus conclude.






                                      share|cite|improve this answer









                                      $endgroup$



                                      Everything you did is good. Indeed, it suffices to show that $I_n:=int_0^1 x^n ln^2 x = frac{2}{(n+1)^3}$ to conclude. The interversion $int / Sigma$ is possible as everything here is positive.



                                      To compute $I_n$, I tried an integration by parts (using that a primitive of $ln^2 x$ is $x(ln^2 x-2ln x)$) to obtain the relation $$I_n = frac{-2}{n+1}int_0^1 x^nln x.$$
                                      Let us call this latter integral $J_n$. Once again with an integration by parts, you can show that $J_n = frac{-1}{(n+1)^2}$, and thus conclude.







                                      share|cite|improve this answer












                                      share|cite|improve this answer



                                      share|cite|improve this answer










                                      answered Aug 4 '16 at 20:55









                                      VincentVincent

                                      293110




                                      293110






























                                          draft saved

                                          draft discarded




















































                                          Thanks for contributing an answer to Mathematics Stack Exchange!


                                          • Please be sure to answer the question. Provide details and share your research!

                                          But avoid



                                          • Asking for help, clarification, or responding to other answers.

                                          • Making statements based on opinion; back them up with references or personal experience.


                                          Use MathJax to format equations. MathJax reference.


                                          To learn more, see our tips on writing great answers.




                                          draft saved


                                          draft discarded














                                          StackExchange.ready(
                                          function () {
                                          StackExchange.openid.initPostLogin('.new-post-login', 'https%3a%2f%2fmath.stackexchange.com%2fquestions%2f1882695%2ffind-int-01-frac-ln21-xx-dx%23new-answer', 'question_page');
                                          }
                                          );

                                          Post as a guest















                                          Required, but never shown





















































                                          Required, but never shown














                                          Required, but never shown












                                          Required, but never shown







                                          Required, but never shown

































                                          Required, but never shown














                                          Required, but never shown












                                          Required, but never shown







                                          Required, but never shown







                                          Popular posts from this blog

                                          Mario Kart Wii

                                          What does “Dominus providebit” mean?

                                          Antonio Litta Visconti Arese